Đến nội dung

Hình ảnh

Số hạng tổng quát dãy không chính phương…

* * * * * 1 Bình chọn

  • Please log in to reply
Chủ đề này có 5 trả lời

#1
hxthanh

hxthanh

    Tín đồ $\sum$

  • Hiệp sỹ
  • 3921 Bài viết
Cho dãy số $(u_n)_{n\ge 1}\; :\;\{3,5,7,8,10,11,13,…\}$
Dãy trên gồm tất cả các số tự nhiên không bao gồm số chính phương và số “phụ phương”. Số “phụ phương” là số tự nhiên dạng $m(m+1)$.
Xác định số hạng tổng quát của dãy $(u_n) $ trên.

Bài viết đã được chỉnh sửa nội dung bởi hxthanh: 08-04-2023 - 23:06


#2
Nobodyv3

Nobodyv3

    Generating Functions Faithful

  • Thành viên
  • 940 Bài viết
Wow, Đề không đụng hàng!
===========
Thà rót cho ta..... trăm nghìn chung... rượu độc ...miễn sao đừng bắt em làm toán!..hu hu...

#3
DOTOANNANG

DOTOANNANG

    Đại úy

  • ĐHV Toán Cao cấp
  • 1609 Bài viết

quater-square


Bài viết đã được chỉnh sửa nội dung bởi DOTOANNANG: 18-10-2023 - 19:13


#4
hxthanh

hxthanh

    Tín đồ $\sum$

  • Hiệp sỹ
  • 3921 Bài viết

Anh Thanh, cho em hỏi một nửa số phụ phương nên gọi là gì vậy ngoài tổng của $k+ 1$ số nguyên không âm đầu tiên? Em đang code FRACTRAN OEIS/A003056 bắt gặp luôn ví dụ của anh.

Đó là số tam giác (triangular number) . Ý nghĩa của nó là số các quả bóng kích thước bằng nhau xếp thành hình bao là một tam giác đều.
Còn số “phụ phương” trên các tài liệu họ gọi là số “phần tư phương” (quarter-square). Không hiểu sao (thực ra anh biết) họ lại gọi như vậy? Anh thấy gọi là phụ phương hay “gần chính phương” hợp lý hơn :D

Bài viết đã được chỉnh sửa nội dung bởi hxthanh: 09-04-2023 - 10:44


#5
chanhquocnghiem

chanhquocnghiem

    Thiếu tá

  • Thành viên
  • 2494 Bài viết

Cho dãy số $(u_n)_{n\ge 1}\; :\;\{3,5,7,8,10,11,13,…\}$
Dãy trên gồm tất cả các số tự nhiên không bao gồm số chính phương và số “phụ phương”. Số “phụ phương” là số tự nhiên dạng $m(m+1)$.
Xác định số hạng tổng quát của dãy $(u_n) $ trên.

$\mathbf{TH1}$ : $n$ là số chính phương ($n=k^2$)

  Dễ thấy rằng $u_n=\left ( \sqrt{n}+1 \right )^2-1=n+2\sqrt{n}=n+2\left \lfloor \sqrt{n} \right \rfloor$.

$\mathbf{TH2}$ : $k^2< n\leqslant k(k+1)$.

  Khi đó $u_n=\left ( \left \lfloor \sqrt{n} \right \rfloor+1 \right )^2+n-\left \lfloor \sqrt{n} \right \rfloor^2=n+2\left \lfloor \sqrt{n} \right \rfloor+1$.

$\mathbf{TH3}$ : $k(k+1)< n< (k+1)^2$.

  Khi đó $u_n=\left ( \left \lfloor \sqrt{n} \right \rfloor+1 \right )^2+n-\left \lfloor \sqrt{n} \right \rfloor^2+1=n+2\left \lfloor \sqrt{n} \right \rfloor+2$.

Kết hợp cả ba trường hợp trên, ta có :

$u_n=n+2\left \lfloor \sqrt{n} \right \rfloor+\left \lfloor \frac{n-\left \lfloor \sqrt{n} \right \rfloor^2+\left \lfloor \sqrt{n} \right \rfloor-1}{\left \lfloor \sqrt{n} \right \rfloor} \right \rfloor$.
 


Bài viết đã được chỉnh sửa nội dung bởi chanhquocnghiem: 09-04-2023 - 12:17

...

Ðêm nay tiễn đưa

Giây phút cuối vẫn còn tay ấm tay
Mai sẽ thấm cơn lạnh khi gió lay
Và những lúc mưa gọi thương nhớ đầy ...

 

http://www.wolframal...-15)(x^2-8x+12)


#6
hxthanh

hxthanh

    Tín đồ $\sum$

  • Hiệp sỹ
  • 3921 Bài viết
Xét dãy $(a_n)_{n\ge 1}\; : \;\{1^2,1.2,2^2,2.3,3^2,3.4,…\}$
Trong đó:
$\begin{cases}a_{2n-1}=n^2 \\ a_{2n}=n(n+1)\end{cases} \Rightarrow\left | \begin{align}\nonumber a_n&=\left\lfloor \frac{n+1}{2}\right\rfloor . \left\lfloor\frac{n+2}{2}\right\rfloor \\ \label{eq1} &=\left\lfloor\frac{(n+1)^2}{4}\right\rfloor\end{align}\right. $
\eqref{eq1} chính là công thức tổng quát của dãy chỉ bao gồm số chính phương và số phụ phương. Tên gọi “phần tư phương” (quarter-square) xuất phát từ đây. Mọi số phụ phương đều biểu diễn được dưới dạng phần nguyên của một phần tư số chính phương
Trở lại với bài toán
Rõ ràng $(u_n)$ và $(a_n)$ là hai dãy “rời nhau”.Xét:
\begin{equation}\label{eq2} a_m<u_n<a_{m+1}\end{equation}
Khi đó ta có $u_n=n+m$, bởi vì $m$ chính là số các số hạng bị loại ra khỏi dãy số tự nhiên.
Thay $m=u_n-n$ vào \eqref{eq2} ta có:
\begin{equation} \left\lfloor\frac{(u_n-n+1)^2}{4}\right\rfloor<u_n\le \left\lfloor\frac{(u_n-n+2)^2}{4}\right\rfloor-1 \label{eq3}\end{equation}
Từ \eqref{eq3} dễ dàng có được:
$n+2\sqrt n \le u_n<n+1+2\sqrt n$
Hay $$u_n=n+\left\lceil 2\sqrt n\right\rceil$$ hoặc $$u_n=n+1+\left\lfloor\sqrt{4n-3}\right\rfloor$$
Nếu không muốn dùng hàm ceilling
——
Lưu ý: Hai kết quả này cùng với kết quả của bạn @chanhquocnghiem không hề mâu thuẫn!




1 người đang xem chủ đề

0 thành viên, 1 khách, 0 thành viên ẩn danh